Bài 8 (tr 118 139) nguyễn minh tuấn

22 104 0
Bài 8  (tr 118 139)  nguyễn minh tuấn

Đang tải... (xem toàn văn)

Tài liệu hạn chế xem trước, để xem đầy đủ mời bạn chọn Tải xuống

Thông tin tài liệu

LỜI GIẢI CHO MỘT LỚP CÁC BẤT ĐẲNG THỨC ĐỒNG BẬC Nguyễn Minh Tuấn Trường Đại học Giáo dục, ĐHQG Hà Nội email: nguyentuan@vnu.edu.vn Ngày tháng 10 năm 2013 Tóm tắt nội dung Trong báo cáo chúng tơi đề cập đến lớp bất đẳng thức đồng bậc quen thuộc tốn phổ thơng hướng mở rộng Bất đẳng thức đồng bậc Sau ta đề cập đến lớp bất đẳng thức thường gặp tốn học, bất đẳng thức đồng bậc Bất đẳng thức đồng bậc có liên quan định đến khái niệm giải tích tốn học hình học đại số, hàm Định nghĩa 1.1 Giả sử k số dương cho trước Hàm số f : Rn −→ R gọi bậc k đẳng thức sau f (tx1 , , txn ) = tk f (x1 , , xn ) với t > Trong phạm vi báo cáo này, f, g hai hàm có bậc k ta gọi bất đẳng thức f (x1 , , xn ) > g(x1 , , xn ), f (x1 , , xn ) ≥ g(x1 , , xn ) bất đẳng thức đồng bậc Nói cách nơm na, bất đẳng thức đồng bậc bất đẳng thức số hạng tham đại lượng (tham biến) có bậc 118 Tuy thế, cần thiết phải nhấn mạnh tùy thuộc vào toán cụ thể mà cách gọi "bất đẳng thức đồng bậc" vừa đề cập có ý nghĩa tương đối Thực vậy, xét ví dụ đơn giản sau Giả sử ba số dương a, b, c thỏa mãn abc = Chứng minh bất đẳng thức a + b + c ≥ Thoạt nhìn, ta khơng nói bất đẳng thức đồng bậc Tuy nhiên, với giả thiết toán này, bất đẳng thức cần chứng minh tương đương với bất đẳng thức sau √ a + b + c ≥ abc Đây lại bất đẳng thức đồng bậc! Nói chung, cách tư nhìn nhận vấn đề có vai trò quan trọng việc cảm nhận giới khách quan xảy xung quanh Bài toán 1.1 Cho a, b, c ba số thực dương Chứng minh aα+β + bα+β + cα+β ≥ aα bβ + bα cβ + cα aβ , (1.1) với α, β ∈ R+ Lời giải Đây bất đẳng thức đồng bậc f (a, b, c) = aα+β + bα+β + cα+β , aα b β + b α c β + c α aβ hai hàm số bậc α + β Áp dụng bất đẳng thức (3.15) suy rộng ta αaα+β + βbα+β ≥ aα b β , α+β α+β αb + βcα+β ≥ bα c β , α+β α+β αc + βaα+β ≥ c α aβ α+β Cộng chiều vế bất đẳng thức ta suy điều phải chứng minh Bài toán hai số α, β âm Thật vậy, đặt > 0, a y : = > 0, b z : = > 0, c x:= 119 γ : = −α > 0, δ : = −β > Khi (1.1) tương đương với xγ+δ + y γ+δ + z γ+δ ≥ xγ y δ + y γ z δ + z γ xδ (1.2) với γ, δ ∈ R+ Bất đẳng thức (1.2) có dạng (1.1) Có câu hỏi tự nhiên đặt ra: bất đẳng thức (1.1) hay khơng α β trái dấu nhau? Điều thú vị (có lẽ ngạc nhiên nữa) là: chiều bất đẳng thức thay đổi hai số mũ α β trái dấu nhau! Trước hết, xét ví dụ sau Bài tốn 1.2 Cho a, b, c ba số thực dương Chứng minh a2 b c + + ≥ a + b + c b c a (1.3) Lời giải Bất đẳng thức toán viết dạng khác sau: a2−1 + b2−1 + c2−1 ≤ a2 b−1 + b2 c−1 + c2 a−1 Như vậy, theo dạng bất đẳng thức (1.1) α = β = −1, chiều bất đẳng thức thay đổi Ta chứng minh bất đẳng thức Áp dụng bất đẳng thức (3.15) ta có a2 + b ≥ 2a b b2 + c ≥ 2b c c2 + a ≥ 2c a Cộng chiều vế ta bất đẳng thức cần chứng minh Bài toán 1.3 Cho a, b, c ba số thực dương Chứng minh √ √ √ b c a 3 √ +√ +√ ≥ a2 + b + c 3 c a b Lời giải Áp dụng bất đẳng thức (3.15) ta có √ √ a 3 √ + ab ≥ a2 b √ √ b 3 √ + bc ≥ b2 c √ √ c 3 √ + ca ≥ c2 a 120 (1.4) Cộng chiều vế ta √ √ √ a b c 3 √ +√ +√ ≥ a2 + b + c 3 c a b √ √ √ √ √ √ 3 3 a2 + b + c − ab + bc + ca + Ta chứng minh √ √ √ √ √ √ 3 3 a2 + b + c − ab + bc + ca ≥ (1.5) Thật vậy, đặt √ a > 0, √ y : = b > 0, √ z : = c > x:= Khi đó, (1.5) trở thành x2 + y + z − (xy + yz + zx) ≥ Bất đẳng thức cuối kết Bài toán 1.1 chứng minh Từ suy √ √ √ c b a + b2 + c √ √ + ≥ a +√ 3 c a b Đây điều phải chứng minh Bài toán 1.4 Cho a, b, c ba số thực dương Chứng minh √ √ √ a b c √ +√ +√ ≥ a + b + c 3 b c a2 (1.6) Lời giải Áp dụng bất đẳng thức (3.15) ta có: √ a √ + b≥2 b a √ , b √ b √ + 3c≥2 c √ c √ + 3a≥2 a b √ , c c √ a Cộng chiều vế ta a b c √ + √ + √ ≥ 3 b c a2 a √ + b b √ + c 121 c √ a − √ a + √ √ b + 3c Bài tốn hồn thành sau ta chứng minh bất đẳng thức sau đây: a √ + b b √ + c √ √ √ c √ ≥ a + b + c a Bất đẳng thức tương đương với: √ √ √ √ √ √ a b c √ +√ +√ ≥ a + b + c 6 c a b Đặt √ a > 0, √ y : = b > 0, √ z : = c > x:= Bất đẳng thức cần chứng minh trở thành √ y z x √ √ + + ≥ x2 + √ 3 y z x y2 + √ z2 Nhưng bất đẳng thức có dạng (1.4) chứng minh Bài tốn 1.3 Từ suy điều phải chứng minh Sau toán tổng quát Bài toán 1.5 Cho a, b, c ba số thực dương α > 0, β > cho trước Chứng minh aα b α c α + β + β ≥ aα−β + bα−β + cα−β (1.7) bβ c a Bất đẳng thức (1.7) viết dạng khác sau: aα+(−β) + bα+(−β) + cα+(−β) ≤ aα b−β + bα c−β + cα a−β Như vậy, chiều bất đẳng thức ngược với chiều (1.1) Trước hết ta xét toán sau: Bài toán 1.6 Chứng minh a, b, c ba số thực dương β ∈ (0, 1) a b c + + ≥ a1−β + b1−β + c1−β β β β b c a 122 (1.8) Lời giải Xét dãy số {sk }k≥1 xác định sau: s1 = , s2 = + s3 = + , + , 1 + + ··· + 1 sk = + + · · · + sk−1 = , 2k−1 1 + 2k−1 2k Chiến thuật chứng minh toán là: trước tiên chứng minh (1.8) với β ∈ [0, s1 ]; sau ta chứng minh (1.8) với β ∈ [0, sk ] với β ∈ [sk , sk+1 ] Vì 2k − =1 k→+∞ 2k lim sk = lim k→+∞ nên ta suy (1.8) với β ∈ (0, 1) Bước Chứng minh (1.8) β ∈ (0, ] Áp dụng bất đẳng thức (3.15) ta có a + a1−2β bβ ≥ 2a1−β , bβ b + b1−2β cβ ≥ 2b1−β , cβ c + c1−2β aβ ≥ 2c1−β β a Cộng chiều bất đẳng thức ta a b c + β + β ≥ a1−β + b1−β + c1−β β b c a + a1−β + b1−β + c1−β − a1−2β bβ + b1−2β cβ + c1−2β aβ Từ giả thiết suy − β > 0, β > 0, − 2β ≥ Do đó, ta áp dụng bất đẳng thức đồng bậc Bài toán 1.1 để thu a1−β + b1−β + c1−β − a1−2β bβ + b1−2β cβ + c1−2β aβ Từ suy a b c + + ≥ a1−β + b1−β + c1−β b β c β aβ Đây điều cần chứng minh 123 ≥ Bước Giả sử (1.8) với β ∈ [0, sk ] Xét cố định β ∈ [sk , sk+1 ] Ta chứng minh (1.8) với giá trị β này, nghĩa ta chứng minh b c a + β + β ≥ a1−β + b1−β + c1−β β b c a (1.9) Áp dụng bất đẳng thức (3.15) ta có √ √ a a 1−β , +b ≥ ab1−2β = √ β 2β b ( b) −1 √ √ b b 1−β +c ≥ bc1−2β = √ 2β −1 , β c ( c) √ √ c c 1−β 1−2β +c ≥ ca = √ 2β −1 β a ( a) Cộng chiều bất đẳng thức ta √ √ √ b c a b c a + β + β ≥2 √ + √ 2β −1 + √ 2β −1 β b c a ( a) ( b)2β −1 ( c) − a1−β + b1−β + c1−β (1.10) Phép chứng minh (1.9) hoàn tất ta chứng minh bất đẳng thức sau: √ √ √ a b c √ + √ 2β −1 + √ 2β −1 ≥ a1−β + b1−β + c1−β (1.11) ( a) ( b)2β −1 ( c) Đặt √ a > 0, √ y : = b > 0, √ z : = c > 0, δ : = 2β − x:= Bất đẳng thức cần chứng minh (1.11) tương đương với x y z + δ + δ ≥ x1−δ + y 1−δ + z 1−δ δ y z x (1.12) Từ giả thiết β ∈ [sk , sk+1 ] dễ dàng suy δ ∈ [0, sk ] Theo giả thiết bước (1.12) đúng, nên (1.11) Từ hai bất đẳng thức (1.10), (1.11) suy (1.9) với β ∈ [0, sk+1 ] Vậy, bất đẳng thức (1.7) với β ∈ (0, 1) lim sk = k→+∞ 124 Bây ta trở Bài toán 1.5 Trước hết, ta cần chứng minh cho trường hợp α ≥ β Thật vậy, α ≤ β ta đặt > 0, a y : = > 0, b z : = > c x:= Khi đó, bất đẳng thức trở thành xβ+(−α) + y β+(−α) + z β+(−α) ≤ y β x−α + z β y −α + xβ z −α Lúc ta lại có β ≥ α Như vậy, ta giả thiết α ≥ β Ta đặt x : = aα > 0, y : = bα > 0, z : = cα > 0, β δ:= α Từ suy δ ∈ (0, 1), β β a = (a ) α = x α = xδ , β β β α α b = (b ) = y α = y δ , β β β α α c = (c ) = z α = z δ , β 1− α−β α a = (a ) α = x1−δ , β 1− α−β α b = (b ) α = y 1−δ , β 1− α−β α c = (c ) α = z 1−δ β α Do đó, bất đẳng thức Bài toán 1.5 trở thành (1.8) Bài toán 1.6: x y z + + ≥ x1−δ + y 1−δ + z 1−δ , δ δ δ y z x δ ∈ (0, 1) Tóm lại, ta phát biểu mệnh đề sau 125 Khẳng định Giả sử a, b, c ba số thực dương α, β ∈ R Khi đó, • Nếu α β số dấu aα+β + bα+β + cα+β ≥ aα bβ + bα cβ + cα aβ • Nếu α β số trái dấu aα+β + bα+β + cα+β ≤ aα bβ + bα cβ + cα aβ Bây ta chuyển sang dạng bất đẳng thức đồng bậc khác Ví dụ 1.1 Giả sử a, b, c ba số thực dương Chứng minh a2 b2 c2 a+b+c + + ≥ b+c c+a a+b (1.13) Lời giải Áp dụng bất đẳng thức (3.15) ta có b+c a2 + ≥ a, b+c b2 c+a + ≥ b, c+a a+b c2 + ≥ c a+b Cộng chiều bất đẳng thức ta suy điều phải chứng minh Ví dụ 1.2 Giả sử a, b, c ba số thực dương bất kỳ, p, q ∈ R+ cho trước Chứng minh a2 b2 c2 a+b+c + + ≥ (1.14) pb + qc pc + qa pa + qb p+q Lời giải Áp dụng bất đẳng thức (3.15) ta có a2 pb + qc a + ≥2 , pb + qc (p + q) p+q b b2 pc + qa + ≥2 , pc + qa (p + q) p+q c2 pa + qb c + ≥ pa + qb (p + q)2 p+q Cộng chiều bất đẳng thức ta suy điều phải chứng minh 126 Bài tập Những toán sau dành cho đọc giả Bài toán 2.1 Giả sử a, b, c ba số thực dương bất kỳ, p, q ∈ R+ cho trước Chứng minh bα cα aα−β + bα−β + cα−β aα , + + ≥ pbβ + qcβ pcβ + qaβ paβ + qbβ p+q với α ≥ β ≥ Bài toán 2.2 Giả sử a, b, c ba số thực dương bất kỳ, p, q ∈ R+ cho trước Chứng minh aα bα cα aα−β + bα−β + cα−β + + ≥ , (pb + qc)β (pc + qa)β (pa + qb)β (p + q)β với α ≥ β ≥ Bài toán 2.3 (mở rộng Khẳng định 1) Giả sử a1 , a2 , , an n số thực dương, α, β ∈ R Chứng minh • Nếu α β số dấu α+β aα+β + a2α+β + · · · + aα+β n−1 + an ≥ aα1 aβ2 + aα2 aβ3 + · · · + aαn−1 aβn + aαn aβ1 • Nếu α β số trái dấu α+β aα+β + a2α+β + · · · + aα+β n−1 + an ≤ aα1 aβ2 + aα2 aβ3 + · · · + aαn−1 aβn + aαn aβ1 Bài toán 2.4 Giả sử a1 , a2 , , an n số thực dương, p, q ∈ R+ cho trước Chứng minh a2n−1 a21 a22 a2n + + ··· + + pa2 + qa3 pa3 + qa4 pan + qa1 pa1 + qa2 ≥ 127 a1 + a2 + · · · + an p+q Phụ lục: Một vài phép chứng minh bất đẳng thức (AM-GM) Xét bất đẳng thức a1 + a2 + · · · + an−1 + an √ ≥ n a1 a2 an−1 an n (3.15) Có nhiều phép chứng minh cho bất đẳng thức (3.15) tiếng Sau đây, xét số phép chứng minh dựa ý tưởng toán học khác Xin tham khảo [10], có nhiều phép chứng minh khác 3.1 Hàm mũ phép chứng minh Polya Trong mục này, sử dụng bất đẳng thức liên quan đến hàm mũ ex để trình bày phép chứng minh Polya1 cho bất đẳng thức (3.15) Bổ đề 3.1 Bất đẳng thức sau ex−1 ≥ x, với x ∈ R Chứng minh Xét hàm số f (x) = ex−1 − x Dễ thấy hàm f khả vi liên tục Ta có f (x) = ex−1 − Ta thấy phương trình f (x) = có nghiệm x0 = Đạo hàm cấp hai f (x) = ex−1 > với x ∈ R Từ suy f (x0 ) = f (1) > Do đó, x0 = điểm cực tiểu (toàn cục) hàm f Bởi vậy, f (x) ≥ f (1) = 0, với x ∈ R Hay, ex−1 ≥ x với x ∈ R Bổ đề chứng minh 128 Hình 1: Hàm số y = ex−1 − x Hình minh họa rõ ràng cho bất đẳng thức Bổ đề 3.1 Thật vậy, đồ thị hàm số y = ex−1 − x thuộc nửa trục hoành, tiếp xúc với trục hoành điểm x0 = Bây ta chứng minh bất đẳng thức (3.15) Đặt A := a1 + a2 + · · · + an n Áp dụng bất đẳng thức Bổ đề 3.1 ta có bất đẳng thức sau: a1 −1 a1 eA ≥ ≥ 0, A a2 −1 a2 eA ≥ ≥ 0, A an−1 −1 an−1 e A ≥ ≥ 0, A an −1 an eA ≥ ≥ A Nhân chiều vế tất bất đẳng thức ta a1 + a2 + · · · + an −n a1 a2 an A e ≥ An Hay en−n = e0 = ≥ G Polya nhà toán học người Hungary 129 a1 a2 an An Bất đẳng thức cuối tương đương với A≥ √ n a1 a2 an Ta nhận bất đẳng thức (3.15) 3.2 Quy nạp tiến-lùi Cauchy Ta chứng minh (3.15) quy nạp theo n Quá trình chứng minh quy nạp chia làm hai bước Bước 1: Quy nạp tiến Với n = ta có (a1 − a2 )2 ≥ Từ suy (a1 + a2 )2 ≥ 4a1 a2 ≥ Khai hai vế Hay √ a1 + a2 ≥ a1 a2 a1 + a2 √ ≥ a1 a2 Bất đẳng thức chứng minh cho trường hợp n = Tiếp theo, ta chứng minh cho trường hợp n = Giả sử a, b, c, d bốn số không âm Áp dụng cho trường hợp n = ta √ √ √ √ √ √ √ a+b+c+d ab + cd ab + cd ≥ = ≥ ab cd = abcd 4 Tiếp tục trình tương tự, ta dễ dàng chứng minh bất đẳng thức (3.15) cho tất trường hợp n lũy thừa 2, nghĩa n = 2k , k ∈ N Bước 2: Quy nạp lùi Giả thiết bất đẳng thức (3.15) với giá trị n0 ≥ đó, nghĩa với n0 số không âm a1 , a2 , , an0 ta có a1 + a2 + · · · + an0 −1 + an0 √ ≥ n0 a1 a2 an0 −1 an0 n0 Ta chứng minh bất đẳng thức (3.15) với n0 − Xét gồm n0 − số không âm a1 ≥ 0, a2 ≥ 0, , an0 −1 ≥ 130 Áp dụng bất đẳng thức (3.15) cho n0 đại lượng, ta có a1 + a2 + · · · + an0 −1 n0 − a1 + a2 + · · · + an0 −1 + = a1 + a2 + · · · + an0 −1 n0 − n0 n0 a1 a2 an0 −1 ≥ a1 + a2 + · · · + an0 −1 n0 − Hay a1 + a2 + · · · + an0 −1 ≥ n0 − n0 a1 a2 an0 −1 a1 + a2 + · · · + an0 −1 n0 − Lũy thừa n0 hai vế bất đẳng thức cuối ta a1 + a2 + · · · + an0 −1 n0 − n0 ≥ a1 a2 an0 −1 a1 + a2 + · · · + an0 −1 n0 − Giản ước hai vế cho đại lượng a1 + a2 + · · · + an0 −1 = 0, n0 − khai (bậc n0 − 1) hai vế ta thu a1 + a2 + · · · + an0 −1 ≥ n0 − √ n0 −1 a1 a2 an0 −1 Như vậy, bất đẳng thức (3.15) với n0 − Theo nguyên lý quy nạp toán học ta suy bất đẳng thức (3.15) với n nhỏ n0 Từ hai bước chứng minh ta suy bất đẳng thức (3.15) với n số a1 , a2 , , an , n ≥ Thực vậy, giả sử n ∈ N cho trước Nếu n = 2k (k ∈ N) (3.15) theo Bước Nếu n lũy thừa tồn số k ∈ N n < 2k := n0 Theo phép chứng minh Bước 1, (3.15) với n0 = 2k , theo Bước 2, (3.15) với n n < n0 Trong phép chứng minh vừa trình bày, người ta nói Bước quy nạp tiến Bước quy nạp lùi Phép quy nạp kiểu Cauchy đề xuất vào năm 1821 Có lẽ thế, phép quy nạp kiểu mang tên nhà toán học Cauchy2 Louis Francois Cauchy nhà toán học người Pháp 131 3.3 Một điều chỉnh quy nạp tiến–lùi Bước phép chứng minh quy nạp tiến-lùi Cauchy trình bày mục trước cho ý tưởng Ký hiệu m := 2k > n Theo Bước phép quy nạp tiến-lùi Cauchy, bất đẳng thức (3.15) cho m số thực không âm Ta chứng minh (3.15) với giá trị n nằm khoảng hai số lũy thừa 2, nghĩa 2k−1 < n < 2k Giả sử x1 , x2 , , xn n số không âm Đặt α := x1 + x2 + · · · + xn n Ta xét thêm m − n số sau: xn+1 = xn+2 = · · · = xm = α Áp dụng bất đẳng thức (3.15) cho m số ta có α= = = = = ≥ = x1 + x2 + · · · + xn n m (x1 + x2 + · · · + xn ) n m m−n (x1 + x2 + · · · + xn ) x1 + x2 + · · · + xn + n m x1 + x2 + · · · + xn + (m − n) α m x1 + x2 + · · · + xn + xn+1 + · · · + xm m √ m x x ···x x n n+1 · · · xm m x1 x2 · · · xn αm−n Ta suy bất đẳng thức đây: αm ≥ x1 x2 · · · xn αm−n , α n ≥ x1 x2 · · · xn , √ α ≥ n x1 x2 · · · xn Bất đẳng thức chứng minh cho n số Do dãy số 2, 4, 8, , 2k , không bị chặn, nên bất đẳng thức (3.15) với n số không âm 132 3.4 Quy nạp tiến thông thường Phép chứng minh thứ Đặt M := √ n a1 a2 an−1 an Khơng tính tổng quát, ta giả thiết a1 a2 an = Thực vậy, vậy, ta thay ak bk := ak M với k = 1, 2, , n vào bất đẳng thức (3.15), bất đẳng thức cần chứng minh tương đương với b1 + b2 + · · · + bn−1 + bn ≥ 1, n đại lượng b1 , b2 , , bn thỏa mãn đẳng thức b1 b2 bn = Sau đây, ta chứng minh bất đẳng thức (3.15) cho trường hợp a1 a2 an = Khi đó, (3.15) có dạng a1 + a2 + · · · + an−1 + an ≥ n (3.16) Dễ dàng chứng minh (3.16) cho trường hợp n = Giả sử (3.16) với n ≥ Xét n + số dương thỏa mãn a1 a2 an an+1 = Do tích chúng nên tồn hai số số không nhỏ số không lớn Khơng tính tổng qt, giả thiết a1 ≥ 1, a2 ≤ Từ suy (a1 − 1)(a2 − 1) ≤ Do a1 + a2 ≥ + a1 a2 Xét n + số: a1 a2 , a3 , , an , an+1 Đương nhiên, n + số thỏa mãn điều kiện a1 a2 a3 an an+1 = 133 (3.17) Theo (3.17) ta có (a1 + a2 ) + (a3 + a4 · · · + an + an+1 ) ≥ + a1 a2 + a3 + a4 + · · · + an + an+1 (3.18) Áp dụng (3.16) cho trường hợp n số vừa nêu ta a1 a2 + a3 + a4 + · · · + an + an+1 ≥ n (3.19) Cộng vế tương ứng (3.18) với (3.19) ta nhận a1 + a2 + · · · + an + an+1 ≥ n + Vậy, bất đẳng thức (3.15) với n ≥ Phép chứng minh thứ hai Đặt µ= x1 + · · · + xn n Bất đẳng thức cần chứng minh tương đương với µ n ≥ x1 x2 · · · xn Đẳng thức thức xảy µ = xi với i = 1, 2, , n Trường hợp n = Bất đẳng thức cần chứng minh tương đương với bất đẳng thức sau (x1 + x2 )2 ≥ 4x1 x2 Hay (x1 − x2 )2 ≥ Bất đẳng thức cuối hiển nhiên Vậy, bất đẳng thức (3.15) cho trường hợp n = Giả sử bất đẳng thức (3.15) với n số không âm (n ≥ 2) Xét n + số thực không âm x1 , x2 , , xn , xn+1 Khi (n + 1)µ = x1 + · · · + xn + xn+1 Nếu tất số xi µ, dễ thấy đẳng thức xảy Trái lại, xét trường hợp tồn số lớn µ số khác nhỏ µ, chẳng hạn xn > µ xn+1 < µ Khi (xn − µ)(µ − xn+1 ) > Bây ta xét n số x1 , , xn−1 , xn , xn := xn + xn+1 − µ ≥ xn − µ > 134 Do nµ = x1 + · · · + xn−1 + xn + xn+1 − µ, = xn nên µ trung bình cộng n số x1 , , xn−1 , xn Theo giả thiết quy nạp, ta có µn+1 = µn · µ ≥ x1 x2 · · · xn−1 xn µ (3.20) Như vậy, (xn + xn+1 − µ)µ − xn xn+1 = (xn − µ)(µ − xn+1 ) > = xn Từ ta suy xn µ > xn xn+1 (3.21) Ta kết luận rằng, số x1 , x2 , , xn−1 khơng, (3.21) có bất đẳng thức thực Nếu vậy, vế phải (3.21) số dương bất đẳng thức nhận cách sử dụng đánh giá (3.20) để có cận vế phải (3.21) Vậy, trường hợp ta có µn+1 > x1 x2 · · · xn−1 xn xn+1 Bất đẳng thức chứng minh 3.5 Cực trị hàm số Bằng cách xây dựng hàm số thích hợp, ta chứng minh (3.15) quy nạp Ta chứng minh (3.15) n = Với n số thực a1 , a2 , , an bất kỳ, ta ký hiệu An : = Gn : = a1 + a2 + · · · + an , n √ n a1 a2 an Giả sử (3.15) với giá trị n đó, nghĩa ta có An ≥ G n với gồm n số thực không âm Xét n + số không âm a1 , a2 , , an , an+1 Ta phải chứng minh bất đẳng thức An+1 ≥ Gn+1 135 (3.22) Xét hàm số f (t) = (an+1 + t) + a1 + a2 + · · · + an n+1 − n+1 (an+1 + t)a1 a2 an Cho tập xác định hàm số khoảng vơ hạn (−an+1 , +∞) Ta có f (t) = 1 − n+1 n+1 n+1 a1 a2 an (an+1 + t)n Phương trình f (t) = có nghiệm t0 = √ n a1 a2 an − an+1 Đạo hàm cấp hàm f biểu thị dạng sau √ n+1 a a a 1 n f (t) = − an+1 + t n+1 n+1 n − n+1 Ta tính đạo hàm cấp hai f (t) = √ n× (n + n+1 1)2 a1 a2 an an+1 + t 2n+1 n+1 Từ ta suy f (t) > với t ∈ (−an+1 , +∞) Cụ thể, ta có f (t0 ) > Do vậy, t0 điểm cực tiểu hàm f tập xác định nó, nghĩa f (t) ≥ f (t0 ), với t ≥ −an+1 Đặc biệt, f (0) ≥ f (t0 ) Dễ dàng tính được: f (0) = An+1 − Gn+1 , f (t0 ) = (an+1 + t0 ) + a1 + a2 + · · · + an n+1 Gn + nAn n − n+1 (an+1 + t0 )a1 a2 an = − Gn = (An − Gn ) n+1 n+1 Như vậy, từ bất đẳng thức f (0) ≥ f (t0 ) ta suy An+1 − Gn+1 ≥ n (An − Gn ) n+1 Theo giả thiết quy nạp bất đẳng thức (3.22) đúng, nghĩa An − Gn ≥ Từ đây, ta suy An+1 − Gn+1 ≥ Bất đẳng thức (3.15) chứng minh 136 3.6 Sử dụng bất đẳng thức Jensen Trong phần xét thêm phép chứng minh bất đẳng thức (3.15) cách sử dụng bất đẳng thức Jensen Nếu chưa biết đến bất đẳng thức Jensen, bạn đọc chuyển sang tìm hiểu bất đẳng thức Chương ?? trước xem xét phép chứng minh Xét hàm f (x) = ln x xác định khoảng (0, +∞) Đạo hàm cấp hai f (x) = −1 < 0, x2 với x ∈ (0, +∞) Từ suy f hàm lõm Áp dụng bất đẳng thức Jensen3 cho hàm lõm ta ln( ln a1 + ln a2 + · · · + ln an−1 + ln an a1 + a2 + · · · + an−1 + an )≥ n n Hay ln( a1 + a2 + · · · + an−1 + an √ ) ≥ ln( n a1 a2 an−1 an ) n Vì hàm ln x đồng biến khoảng (0, +∞) nên a1 + a2 + · · · + an−1 + an √ ≥ n a1 a2 an−1 an n Bất đẳng thức (3.15) chứng minh cho trường hợp tất số a1 , , an dương Hiển nhiên, n số nói có số khơng (3.15) Liên quan đến phép chứng minh vừa nêu, thực không thiết phải chọn hàm số f (x) = ln x trên, mà ta chọn hàm số logarit với số a > 0: f (x) = loga x xác định nửa trục dương Thật vậy, hàm số f lõm đồng biến với a > 1, lồi nghịch biến với a < khoảng xác định nó, ta áp dụng bất đẳng thức Jensen theo cách hoàn toàn tương tự trên, xem Hình Tài liệu [1] Recent Progress in inequality (G V Milovanovic, ed.), Mathematics and Its Applications, Kluwer Academic Publishers, Dordrecht-Boston-London, first ed., 1997 [2] P J Bushell, Shapiro’s cyclic sum, Bull London Math Soc 26 (1994), 564–574 [3] P J Bushell and J B McLeod, Shapiro’s cyclic inequality for even n, J of Inequal & Appl (2002), no 3, 331–348 Johan Jensen nhà tốn học người Đan Mạch 137 Hình 2: Hàm số y = ln(x) [4] V Cirtoaje, On a proof of an inequality of two real numbers, JNSA, V (2011), Issue 2, 130–137 [5] P H Diananda, A cyclic inequality and an extension of it II, Proc Edinburgh Math Soc (2), V 13 (1962), pp 143-152 [6] P H Diananda, On a cyclic sum, Proc Glasgow Math Assoc (1963), 11–13 [7] V G Drinfeld, A cyclic inequality, Math Notes (1971), 68–71 [8] R P Lewwis, Antisocial dinner parties, Fibonacci Quart., to appear [9] M J Lighthill, An invalid inequality, Amer Math Monthly 63 (1956), 191–192 [10] N V Mau, Bất đẳng thức Định lý áp dụng, NXB Giáo dục, Hà Nội–Đà Nẵng, 2006 [11] L J Mordell, On the inequality nr=1 xr /(xr+1 + xr+2 ) ≥ n/2 and some others, Abh Math Se Univ Hamburg 22 (1958), 229–240 [12] , Note on the inequality nr=1 xr /(xr+1 + xr+2 ) ≥ n/2 and some others, J London Math Soc 37 (1962), 176–178 [13] A M Nesbitt, Problem 15114, Educational Times, (1903), 37–38 [14] P Nowosad, Isoperimetric eigenvalue problems in algebras, Comm Pure Appl Math., 21 (1968), pp 401-465 [15] B G Pachpatte, Mathematical Inequalities, North-Holland Mathematical Library, V 67 (first ed.) Amsterdam, The Netherlands: Elsevier, 2005 [16] R A Rankin, A cyclic inequality, Proc Edinburgh Math Soc (2), V 12 (1961), pp 139-147 [17] H S Shapiro, Advanced problem # 4603, Amer Math Monthly, 61 (1954), p 571 138 [18] N E Steenrod, P R Halmos, A M Schiffer, and J A Dieudonné, How to write mathematics, Amer Math Society, USA, 1973 [19] B A Troesch, The validity of shapiro’s cyclic inequality, Math Comp 53 (1989), 657–664 [20] B A Troesch, The shooting method applied to a cyclic inequality, Math Comp., V 34, (1980), pp 175-184 [21] N M Tuan and L Q Thuong, On an extension of shapiro’s cyclic inequality, J Inequalities Appl (2009), no 12, 1–5 [22] N M Tuấn, Cơ sở lý thuyết hàm lồi bất đẳng thức cổ điển, NXB Giáo dục, Tập I, II, 2013 139 ... thuật chứng minh toán là: trước tiên chứng minh (1 .8) với β ∈ [0, s1 ]; sau ta chứng minh (1 .8) với β ∈ [0, sk ] với β ∈ [sk , sk+1 ] Vì 2k − =1 k→+∞ 2k lim sk = lim k→+∞ nên ta suy (1 .8) với β... thức cần chứng minh trở thành √ y z x √ √ + + ≥ x2 + √ 3 y z x y2 + √ z2 Nhưng bất đẳng thức có dạng (1.4) chứng minh Bài toán 1.3 Từ suy điều phải chứng minh Sau toán tổng quát Bài toán 1.5 Cho... minh bất đẳng thức Áp dụng bất đẳng thức (3.15) ta có a2 + b ≥ 2a b b2 + c ≥ 2b c c2 + a ≥ 2c a Cộng chiều vế ta bất đẳng thức cần chứng minh Bài toán 1.3 Cho a, b, c ba số thực dương Chứng minh

Ngày đăng: 03/05/2018, 12:30

Tài liệu cùng người dùng

  • Đang cập nhật ...

Tài liệu liên quan